29.03.2013 Aufrufe

IPhO-Aufgabensammlung Inhaltsverzeichnis

IPhO-Aufgabensammlung Inhaltsverzeichnis

IPhO-Aufgabensammlung Inhaltsverzeichnis

MEHR ANZEIGEN
WENIGER ANZEIGEN

Erfolgreiche ePaper selbst erstellen

Machen Sie aus Ihren PDF Publikationen ein blätterbares Flipbook mit unserer einzigartigen Google optimierten e-Paper Software.

(b) Sei also die Erde ein Inertialsystem und die Erdrotation folglich nicht vorhanden, dann<br />

gilt nach dem 3. Keplerschen Gesetz:<br />

a1<br />

a2<br />

2<br />

=<br />

T1<br />

T2<br />

2<br />

wobei die Indizes zwei verschiedene<br />

geschlossene Ellipsenbahnen um die Erde bezeichnen und T die Umlaufdauern und a die<br />

großen Halbachsen bedeuten. Eine Gerade, die beide Brennpunkte verbindet ist eine gestörte<br />

Form der Ellipse und sein nun mit Index 1 behaftet. Ein Kreis der direkt um den Erdradius<br />

geht ist auch eine gestörte Form der Ellipse und sei nun mit Index 2 behaftet. Dann ist<br />

a2 = rE; T2 = 2π 1 rE<br />

ω = 2π , wobei v hier die erste kosmische Geschwindigkeit ist und<br />

<br />

v1<br />

somit T2 = 4π2 r3 E<br />

GmE . T1 soll ein Jahr sein (Beachte es ist eine Näherung, da der andere<br />

Brennpunkt mit dem Erdmittelpunkt zusammenfällt). Dann ergibt sich:<br />

h ≈ a1 = 3<br />

<br />

T 2 GmE<br />

1<br />

4π2r3 r<br />

E<br />

3 <br />

3<br />

E = T 2 GmE<br />

1 4π2 ≈ 2, 15 · 109m v<br />

v2 = 1 − rE<br />

h ≈ 0, 9985. v ist also unwesentlich kleiner als v2. Es geht auch anders indem<br />

man im wesentlichen Kepler 3 nachrechnet: v(x) =<br />

a<br />

Epot(r → ∞) = 0 sei. Somit ist: 1 y = <br />

rE<br />

dx<br />

2m/(mGmE( 1<br />

x<br />

2<br />

m (Ekin;0 + Epot;0(x) − Epot(x)), wobei<br />

1<br />

− a)) , wobei Ekin;0+Epot;0 = Epot(a),<br />

wobei a der Ort ist wo der Körper umkehrt, sodass Ekin;0(a) = 0 ist. Dieses Integral kann<br />

mit der Substitution x = a sin φ gelöst werden.<br />

Aufgabe 106: Masse und Feder<br />

Erste Phase: Beschleunigung der Masse 3m. Da Zwangsbedingung und 3. Newtonsche Gesetz dazu<br />

führt, dass die Masse m an Ort und Stelle verbleibt (Zwangsbedingung wirkt als Impulsreservoir).<br />

Deshalb gibt es eine Beschleunigung des Schwerpunktes. Die Schwerpunktsenergie ist 3<br />

4 der<br />

Gesamtenergie. Diese Steckt also in einer Translationsenergie. Für die Schwingung verbleibt also<br />

noch ein viertel der Gesamtenergie. Das heißt Ê′ pot = 1<br />

4 Êpot, wobei Ê′ pot die maximale potentielle<br />

Energie der Feder nach dem Stoß ist. Daraus folgt ˆ l ′2 = 1<br />

4 ˆ l 2 ⇒ ˆ l ′ = 1<br />

2 ˆ l = 2 cm.<br />

Aufgabe 107: Houston, wir haben ein Problem<br />

<br />

2<br />

mG<br />

(a) Bernoulli liefert: v = ρ(t) p(t). Da pV = nRT = const und V = ρ ergibt sich: p<br />

ρ =<br />

RT = const. V bezeichnet hierbei das Volumen des Gases, wobei sich gedacht wird,<br />

MLuft<br />

dass es sich insgesamt isotherm ausdehnt. (Das das außerhalb der Raumsphäre nicht der<br />

Fall ist<br />

<br />

beeinflusst den isothermen Prozess innerhalb der Raumsphäre nicht). Somit ist<br />

v = 2 RT<br />

MLuft =<br />

<br />

2kT<br />

RT RT mi ˙<br />

= const. ˙p = ˙ρ = mLuft MLuft MLuft VK , wobei VK das Volumen der<br />

Raumkapsel ist, welches sich nicht ändert. Das Gas im Raumschiff bewegt sich nicht, dehnt<br />

sich langsam aus, lediglich das Gas, dass direkt am Loch ist wird durch den Druckgradienten<br />

beschleunigt. Da sich die Dichte des Gases in der Raumkapsel nur dadurch ändert, das<br />

Gas am Loch mit Dichte ρ(t) ausgestoßen wird, muss die Änderung ρ(t) der Gasmenge,<br />

die ausgestoßen wird, während des Austoßes derselben nicht berücksichtigt werden. mi ˙ =<br />

−Avρ(t) = −AvpMLuft<br />

RT = − ApMLuft<br />

<br />

2RT<br />

RT . Somit ergibt sich:<br />

MLuft<br />

75

Hurra! Ihre Datei wurde hochgeladen und ist bereit für die Veröffentlichung.

Erfolgreich gespeichert!

Leider ist etwas schief gelaufen!